An average force of magnitude 300 N exerted to compress a spring by 20 cm compute the work done by the force​

Answers

Answer 1

Answer:  60 Joules (J)

Explanation:

To compute the work done by a force, you can use the formula:

Work = Force × Distance × cosθ

In this case, the force applied is 300 N, and the distance is 20 cm (which can be converted to meters by dividing by 100, giving us 0.20 m).

The angle θ represents the angle between the direction of the force and the direction of displacement. Since the force is used to compress the spring, the displacement is in the same direction as the force, and the angle θ is 0 degrees. The cosine of 0 degrees is 1, so we can omit the cosine term.

Now we can calculate the work done:

Work = 300 N × 0.20 m

Work = 60 Joules (J)

Therefore, the work done by the force in compressing the spring is 60 Joules (J).


Related Questions

What are regular and irregular reflection of light? plz help its
urgent..​

Answers

Explanation:

Regular reflection: It is the reflection from a smooth surface such that the light rays are evenly parallel to each other and an image is formed. ... Irregular reflection: It is the diffused reflection from uneven surface such that the light rays are not parallel to each other and do not form an image.

[BWS.02]If the same experiment is repeated in different parts of the world by different scientists,

the results will be the same
the results will become invalid
the outcome of the experiment will be non testable
the outcome of the experiment will be non observable

Answers

Answer:

the results will be the same.it may be

If the same experiment is repeated in different parts of the world by different scientists, the results will be the same.

What is scientific experiment?

An experiment is a procedure that is carried out to support or refute a hypothesis, or to determine the efficacy or likelihood of something that has never been tried before. Experiments shed light on cause-and-effect relationships by demonstrating what happens when a specific factor is changed.

Controls are typically included in experiments to minimise the effects of variables other than the single independent variable. This improves the reliability of the results, often by comparing control measurements to the other measurements. Scientific controls are an essential component of the scientific method.

Hence, If the same experiment is repeated in different parts of the world by different scientists, the results will be the same.

Learn more about experiment here:

https://brainly.com/question/11256472

#SPJ2


The string will break if the tension in
it exceeds 0.180 N. What is the
smallest possible value of d (in cm)
before the string breaks?

Answers

Answer:

define d first?

you need to list more variables

Answer:

list more valuable unit

An ordinary ruler is used to measure the area and its error of a rectangle. It is found that their sides are 5.0 cm long and 2.0 cm width. The error in area (in cm) is​

Answers

Answer:

You need to know the accuracy to which you can read the ruler:

Suppose that you can read the read the ruler to the nearest milimeter

A = L * W     your calculated area of the rectangle

A + ΔA = (L + ΔL) * (W + ΔW) = L W + L ΔW + W * ΔL + ΔL ΔA

Or ΔA =  L ΔW + W ΔL

Where we have subtracted A = L * W and the term ΔL * ΔA is very small

So (5 + .1) * (2 + .1) - 5 * 2 = .1 * 2 + .1 * 5 = .7 cm^2

Then you report A = 10 cm^2 +- .7 cm^2    including the - sign for completeness

What is the acceleration of a .3 kg mass when there is a net force of 25.9 N on it?

Answers

Answer:

86.33m/s^2

Explanation:

Acceleration = Force/Mass

= 25.9/0.3

= 86.33

As mentioned in the text, the tangent line to a smooth curve r(t) = ƒ(t)i + g(t)j + h(t)k at t = t0 is the line that passes through the point (ƒ(t0), g(t0), h(t0)) parallel to v(t0), the curve’s velocity vector at t0. In Exercises 23–26, find parametric equations for the line that is tangent to the given curve at the given parameter value t = t0.

Answers

Answer:

[tex]x = t[/tex]

[tex]y = \frac{1}{3}t[/tex]

[tex]z =t[/tex]

Explanation:

Given

[tex]r(t) = f(t)i + g(t)j + h(t)k[/tex] at [tex]t = 0[/tex]

Point: [tex](f(t0), g(t0), h(t0))[/tex]

[tex]r(t) = ln\ t_i + \frac{t-1}{t+2}j + t\ ln\ tk[/tex], [tex]t0 = 1[/tex] -- Missing Information

Required

Determine the parametric equations

[tex]r(t) = ln\ ti + \frac{t-1}{t+2}j + t\ ln\ tk[/tex]

Differentiate with respect to t

[tex]r'(t) = \frac{1}{t}i +\frac{3}{(t+2)^2}j + (ln\ t + 1)k[/tex]

Let t = 1 (i.e [tex]t0 = 1[/tex])

[tex]r'(1) = \frac{1}{1}i +\frac{3}{(1+2)^2}j + (ln\ 1 + 1)k[/tex]

[tex]r'(1) = i +\frac{3}{3^2}j + (0 + 1)k[/tex]

[tex]r'(1) = i +\frac{3}{9}j + (1)k[/tex]

[tex]r'(1) = i +\frac{1}{3}j + (1)k[/tex]

[tex]r'(1) = i +\frac{1}{3}j + k[/tex]

To solve for x, y and z, we make use of:

[tex]r(t) = f(t)i + g(t)j + h(t)k[/tex]

This implies that:

[tex]r'(1)t = xi + yj + zk[/tex]

So, we have:

[tex]xi + yj + zk = (i +\frac{1}{3}j + k)t[/tex]

[tex]xi + yj + zk = it +\frac{1}{3}jt + kt[/tex]

By comparison:

[tex]xi = it[/tex]

Divide by i

[tex]x = t[/tex]

[tex]yj = \frac{1}{3}jt[/tex]

Divide by j

[tex]y = \frac{1}{3}t[/tex]

[tex]zk = kt[/tex]

Divide by k

[tex]z = t[/tex]

Hence, the parametric equations are:

[tex]x = t[/tex]

[tex]y = \frac{1}{3}t[/tex]

[tex]z =t[/tex]

Earth's magnetic field is approximately 1/2 gauss, that is 50 micro-tesla because the SI field unit of a tesla is 10,000 gauss. Earth's north geographic pole is close to its south magnetic pole, and magnetic field is directed from the north to the south poles of a magnetic dipole so it goes from Earth's south geographic pole towards its north. Suppose you have wire carrying a large DC current from the south wall of a building to its north wall and that it is horizontal, on the floor. If Earth's field is parallel to the ground and does not dip, what force if any would the wire experience

Answers

Answer:

F = 0

Explanation:

The magnetic force is described by two expressions

for a moving charge

          F = q v x B

for a wire with a current

         F = I L xB

bold indicates vectors

let's write this equation in module form

         F = I L B sin θ

where the angle is between the direction of the current and the direction of the magnetic field

In this case they indicate that the cable goes from the South wall to the North wall, so this is the direction of the current

The magnetic field of the Earth goes from the south to the north and in this part it is horizontal

Therefore the current and the magnetic field are parallel, the angle between them is zero

           sin 0 = 0

consequently the magnetic force is zero

            F = 0

a 2,400 kg car drives north towad a 60kg shopping cartthat has a velocity of zero the two objects collide giving the car a final velocity 4.33m/s north and the shopping cart 8.88m/s north what is the in itial velocity of the car

Answers

Answer:

4.552m/s

Explanation:

[tex]V=\frac{m_{1}v_{1}+m_{2}v_{2}}{m_{1} } =\frac{2400*4.33+60*8.88}{2400}=4.552m/s[/tex]

what element is produced when a gold nucleus loses a proton?

Answers

The element is Platinum.
Hello, it’s Platinum.

Can someone please help, ty!!
Will mark brainliest.

Answers

Answer:

4. unbalanced and Accelerating

5. balance and rest

As every amusement park fan knows, a Ferris wheel is a ride consisting of seats mounted on a tall ring that rotates around a horizontal axis. When you ride in a Ferris wheel at constant speed, what are the directions of your acceleration and the normal force on you (from the always upright seat) as you pass through (a) the highest point and (b) the lowest point of the ride

Answers

Answer:

Answer is explained in the explanation section below.

Explanation:

In this question, we are asked to find out the direction of acceleration and direction of the normal force acting upon us from the always upright seat.

a) You pass through the highest point:

When we sit in the Ferris wheel at the any amusement park, and when it starts rotating and the time when we reach the highest point, then the direction of of our acceleration will be towards the center or it will be towards downward direction.

And at the highest point on the Ferris Wheel, the direction of the normal force F acting upon us will be upwards.

b) You pass through the lowest point of the ride:

When we sit in the Ferris wheel at the any amusement park, and when it starts rotating and the time when we reach the lowest point, then the direction of of our acceleration will be towards the center or it will be towards upward direction.

And at the lowest on the Ferris Wheel, the direction of the normal force F acting upon us will be upwards again.

In picture 1, heat is flowing from the ____ to the _____ In picture 2, heat is flowing from the _______ to the ____​

Answers

Answer: In picture 1, heat is flowing from the liquid to the air. In picture 2, heat is flowing from the air to the liquid

Explanation:

I don't know if I answered correctly, if not I can provide another answer

A point charge q is located at the center of a spherical shell of radius a that has a charge −q uniformly distributed on its surface. Find the electric field for the following points: (a) for all points outside the spherical shell E = keq2/r2 E = q/4πr2 none of these E = keq/r2 E = 0 (b) for a point inside the shell a distance r from the center E = keq2/r2 E = keq/r2 E = 0 E = q/4πr2 none of these

Answers

Answer:

a) E = 0

b) [tex]E = \dfrac{k_e \cdot q}{ r^2 }[/tex]

Explanation:

The electric field for all points outside the spherical shell is given as follows;

a) [tex]\phi_E = \oint E \cdot dA = \dfrac{\Sigma q_{enclosed}}{\varepsilon _{0}}[/tex]

From which we have;

[tex]E \cdot A = \dfrac{{\Sigma Q}}{\varepsilon _{0}} = \dfrac{+q + (-q)}{\varepsilon _{0}} = \dfrac{0}{\varepsilon _{0}} = 0[/tex]

E = 0/A = 0

E = 0

b) [tex]\phi_E = \oint E \cdot dA = \dfrac{\Sigma q_{enclosed}}{\varepsilon _{0}}[/tex]

[tex]E \cdot A = \dfrac{+q }{\varepsilon _{0}}[/tex]

[tex]E = \dfrac{+q }{\varepsilon _{0} \cdot A} = \dfrac{+q }{\varepsilon _{0} \cdot 4 \cdot \pi \cdot r^2}[/tex]

By Gauss theorem, we have;

[tex]E\oint dS = \dfrac{q}{\varepsilon _{0}}[/tex]

Therefore, we get;

[tex]E \cdot (4 \cdot \pi \cdot r^2) = \dfrac{q}{\varepsilon _{0}}[/tex]

The electrical field outside the spherical shell

[tex]E = \dfrac{q}{\varepsilon _{0} \cdot (4 \cdot \pi \cdot r^2) }= \dfrac{q}{4 \cdot \pi \cdot \varepsilon _{0} \cdot r^2 }= \dfrac{q}{(4 \cdot \pi \cdot \varepsilon _{0} )\cdot r^2 }[/tex]

[tex]k_e= \dfrac{1}{(4 \cdot \pi \cdot \varepsilon _{0} ) }[/tex]

Therefore, we have;

[tex]E = \dfrac{k_e \cdot q}{ r^2 }[/tex]

. [30%] We first showed that The electric field for a point charge radiating in 3-dimensions has a distance dependence of 1/r 2 (see Equation 1). In Problem 1 you showed that the electric field for a point charge radiating in 2-dimensions has a distance dependence of 1/r . Consider again the 2-dimensional case described in Problem 1. What distance dependence do you expect for the electric potential

Answers

Answer:

Answer is explained in the explanation section below.

Explanation:

Note: This question is incomplete and lacks necessary data to solve. As it mentioned the reference of problem number 1, which is missing in this question. However, I have found that question on the internet and will be solving the question accordingly.

Solution:

The relation between electric field and the electric potential is:

E = [tex]\frac{dV}{dr}[/tex]

So, making dV the subject, we have:

dV = E x dr

Integrating the above equation, we get.

V = [tex]\int\limits^_ {} \,[/tex]E x dr      Equation 1

Now, in 2-D

E is inversely proportional to the radius r.

E ∝ 1/r

So, we can write: replacing E ∝ 1/r in the equation 1

V ∝  [tex]\int\limits^_ {} \,[/tex][tex]\frac{1}{r}[/tex] x dr

Which implies that,

V ∝  log (r)

Hence, distance dependence expected for the electric potential =  ln (r)

How do objects with the same charger interact

Answers

The interaction between two like-charged objects is repulsive. ... Positively charged objects and neutral objects attract each other; and negatively charged objects and neutral objects attract each other.

Answer:

they repel with each other. object of like charges repel while object of opposite charges attracts with each other.

The carts are moving on a level, frictionless track. After the collision all three carts stick together. Find the speed of the combined carts after the collision.

Answers

Answer:

0.13 m/s

Explanation:

Unfortunately, I don't have an explanation but I guessed the correct answer.

Fill in the question

Answers

4) 55m
5) 30 seconds
6) 1.83m/s

The voltage v(t) = 141.4 cos (ωt) is applied to a load consisting of a 10Ω resistor in parallel with an inductive reactance XL=ωL = 3.77Ω. Calculate the instantaneous power absorbed by the resistor and by the inductor. Also calculate the real and reactive power absorbed by the load, and the power factor. Draw all the voltage, current and power waveforms, also the draw the circuit and phasor diagrams.

Answers

Answer:

A) P(t) = 2651.25 [ 1 - cos2wt ] W

B)  Real power = 999.79 watts

    Reactive power = 2652.86 VA

c) power factor = 0.3526

Explanation:

Given data:

V(t) = 141.4 cos (ωt)

R(t) = 10 Ω

Inductive reactance XL = ωL = 3.77 Ω

Ir(t) = V(t) / R(t) = 14.14

A) Calculate the instantaneous power absorbed by the resistor and by inductor

By resistor :

Pr(t) = V(t) * Ir(t) = 141.4 * 14.14 [tex]cos^{2} wt[/tex] = 1999.396 [tex]cos^{2} wt[/tex]

      hence Pr  = 999.698 (cos2ωt + 1) w

By Inductor :

Pl(t) = V(t) I'L(t) = 141.4 cosωt * 37.5 cos(ωt - 90)  

                        =  5302.5 [tex]sin^2 wt[/tex]

Hence Pl(t) = 5302.5 [tex]sin^2 wt[/tex]   w  =  2651.25 [ 1 - cos2wt ] W

B) calculate the real and reactive power

First we have to determine the power factor

Given that : V(t) = 141.4 cosωt  v ,   Ir(t) = 14.14 cosωt A

IL(t) = 37.5 cos (ωt - 90° )

The phasor representation of the above is :

V = [tex]\frac{141.4}{\sqrt{2} } <0^{0} v[/tex] = 141.4 ∠0° ,  Ir = 10 ∠ 0° , IL = 37. 5 ∠ -90°

Total load current = Ir + IL = 28.35 ∠ -69.35°

power factor = cos -69.35° = 0.3526

Next we will determine the Real and reactive power using the relation below

S = VI = 100 ∠ 0°  * 28.35 ∠ -69.35°

         = 2835 ∠ 69.35°

S = P + jQ = 999.79 + 2652.85 j

Real power  = 999.79 watts

Reactive power = 2652.85 VA

A rope, attached to a weight, goes up through a pulley at the ceiling and back down to a worker. The worker holds the rope at the same height as the connection point between the rope and weight. The distance from the connection point to the ceiling is 40 ft. Suppose the worker stands directly next to the weight (i.e., a total rope length of 80 ft) and begins to walk away at a constant rate of 3 ft/s. How fast is the weight rising when the worker has walked:

Answers

Complete question is;

A rope, attached to a weight, goes up through a pulley at the ceiling and back down to a worker. The worker holds the rope at the same height as the connection point between the rope and weight. The distance from the connection point to the ceiling is 40 ft. Suppose the worker stands directly next to the weight (i.e., a total rope length of 80 ft) and begins to walk away at a constant rate of 3 ft/s. How fast is the weight rising when the worker has walked:

A) 10 feet

B) 30 feet

Answer:

A) 0.728 ft/s

B) 1.8 ft/s

Explanation:

Let the the position of the worker in ft be denoted by s.

Since he begins to walk away at a constant rate of 3 ft/s, then;

ds/dt = 3 ft/s

Now, the rope will form a triangle, with width "s" and the height 40. Since distance from the connection point to the ceiling = 40 ft

Using pythagoras theorem, we can find the length of the rope on this side of the pulley.

Hence, the length of rope on this side of the pulley = √(s² + 40²)

Meanwhile, on the other side the length will be;

(80) - √(s² + 40²)

Also, height of the weight will be;

h = 40 - ((80) - √(s² + 80²))

h = √(s² + 80²) - 40

Differentiating this, we have;

dh/dt = (ds/dt) × (s/√(s² + 40²))

From earlier, we saw that ds/dt = 3 ft/s

Thus;

dh/dt = 3s/√(s² + 40²)

A) when he has walked 10 ft, it means that s = 10. Thus;

dh/dt = (3 × 10)/√(10² + 40²)

dh/dt = 0.728 ft/s

B) when he has walked 30 ft, it means that s = 30. Thus;

dh/dt = (30 × 3)/√(30² + 40²)

dh/dt = 1.8 ft/s

PLEASE HELP ASAP! WILL GIVE BRAINLIEST TO CORRECT ANSWER! HELP!! HELP!!
The diagram shows the structure of an animal cell.



The image of an animal cell is shown with some organelles labeled numerically from 1 to 6. The outer double layer boundary of the cell is labeled 1. A stacked disc like structure is labeled 2. A broad rod shaped structure with an irregular shape inside it is labeled 3. The entire plain section that forms the background of the cell and is within the outer boundary is labeled 4. A small circular shape within the large circular shape is labeled 5. The large central circular shape is labeled 6.


Which number label represents the cell membrane?


1

2

4

6

(this is middle school science)

Answers

Answer:

1. cell membrane

2. golgi body

3. mitochondrion

4. cytoplasm

5. nucleolus

6. nucleus

Explanation:

The correct answer to this question is Option A; 6.

Why?

In a plant cell, the nucleus surrounds the nucleolous, which would be number 5. Therefore, number 6 would be your correct answer.

~Thank you~

A toy car can go 5 mph. How long would it take to go 12 miles?

Answers

60 or 1 hour because 5 times 12 equals 60

please help me I'm begging you Define and give examples of elements and compounds the structure of atoms (electrons, neutrons, and protons)

Answers

An element is a substance whose atoms all have the same number of protons: another way of saying this is that all of a particular element's atoms have the same atomic number. ... When atoms of the same element have different numbers of neutrons, they are called isotopes. Examples of Compounds:
Water - Formula: H2O = Hydrogen2 + Oxygen. ...
Hydrogen Peroxide - Formula: H2O2 = Hydrogen2 + Oxygen2 ...
Salt - Formula: NaCl = Sodium + Chlorine. ...
Baking Soda - Formula: NaHCO3 = Sodium + Hydrogen + Carbon + Oxygen3 ...
Octane - Formula: C8H18 = Carbon8 + Hydrogen18 Atoms consist of three basic particles: protons, electrons, and neutrons. The nucleus (center) of the atom contains the protons (positively charged) and the neutrons (no charge). The outermost regions of the atom are called electron shells and contain the electrons (negatively charged). The electron is a subatomic particle, symbol e⁻ or β⁻ , whose electric charge is negative one elementary charge. Electrons belong to the first generation of the lepton particle family, and are generally thought to be elementary particles because they have no known components or substructure.

A transformer has 150 turns in the primary coil and 350 turns in its secondary coil. If the primary coil has a voltage of 200 volts, how many volts will the secondary coil have?
242 volts
288
353
467

Answers

Answer:

467 volts

Explanation:

Vs/Vp = Ns/Np

Vs = Ns/Np × Vp

Vs = 350/150 × 200 = 7/3 × 200

Vs = 467 volts

In a certain region of space the electric potential increases uniformly from east to west and does not vary in any other direction. The electric field:Group of answer choicespoints east and varies with positionpoints east and does not vary with positionpoints west and varies with positionpoints west and does not vary with positionpoints north and does not vary with position

Answers

Answer:

Explanation:

The relation between electric field and potential difference is as follows

E = - dV / dr

That means if dV is positive , E is negative . In other words , if potential increases , E is negative or in opposite direction in which potential increases .

Here the electric potential increases uniformly from east to west , that means electric field is from west to east . Since potential is uniformly increasing that means

dV / dr = constant

E = constant

Electric field is constant .

So the option which is correct is

" points east and does not vary with position " .

A vibrating object produces periodic waves with a wavelength of 53 cm and a frequency of 15 Hz. How fast do these waves move away from the object?

Answers

Answer:

v = 7.95 m/s

Explanation:

Given that,

Wavelength of a wave, [tex]\lambda=53\ cm=0.53\ m[/tex]

Frequency of a wave, f = 15 Hz

We need to find the speed of the wave. The speed of a wave is given by :

[tex]v=f\lambda\\\\v=15\ Hz\times 0.53\ m\\\\v=7.95\ m/s[/tex]

So, the wave move with a speed of 7.95 m/s.

In an experiment similar to the one pictured below, an electron is projected horizontally at a speed vi into a uniform electric field pointing up. The magnitude of the total vertical deflection, ye, of the electron is measured to be 1 mm. The same experiment is repeated with a proton (whose mass is 1840 times that of the electron) that is also projected horizontally at a speed vi into the same uniform electric field. What is the magnitude of the total vertical deflection, yp, for the proton

Answers

I think you need Graph to figure it out

Using Newton's second law and kinematic projectile motion we can find the proton deflection y = 5.43 10⁻⁷ m, in the opposite direction to the electron deflection.

given parameters

The deflection of the electorn    y₁ = 1 mm = 0.001 m The initial velocity of the electron and proton v_i The mass of the proton m_p = 1840 me

to find

deflection of the proton

For this exercise we will use Newton's second law where the force is electric

            F = ma

            F = q E

where F is the force, q the charge, E the electric field, m the mass and the acceleration of the particle

           q E = m a

           a = q / m E

This acceleration is the direction of the electric field that is perpendicular to the initial velocity (v_i)

Having the acceleration we can use the kinematics relations

If we make the direction of the initial velocity coincide with the x-axis

             v_i = cte

             v_i = x / t

             t = x/ v_i

       

on the y-axis is in the direction of the electric field

            y = v_{iy}  t + ½ a t²

on this axis the initial velocity is zero

            y = [tex]\frac{1}{2} (\frac{q}{m} E) \ t^2[/tex]

subtitute

            y =            (1)

Electron motion.

Let us propose the expression for the electron situation, the length of the displacement must be the same for electron and proton, suppose that it is x = L

In this case the charge q = -e and the mass m = m_e

its substitute in  equation 1

            y₁ = [tex]\frac{1}{2} \ ( \frac{-e}{m_e} E) \ \frac{x^2}{v_i^2}[/tex]  

where y₁, is the lectron deflection.

Proton motion

Between the proton and the electron we have some relationships

          q_p = -e

          m_ = 1840 m_e

we substitute in the equation  1

         y₂ = ½ e / 1840 me E x² / vi²

         y₂ =

         y₂ = - y₁ / 1840

         y₂ = - 0.001 / 1840

         y₂ = - 5.43 10⁻⁷ m

The negative sign indicates that the deflection of the proton is in the opposite direction to the deflection of the electron.

In conclusion they use Newton's second law and kinematics we can find the proton deflection is y = 5.43 10⁻⁷ m

learn more about electric charge movement here:  https://brainly.com/question/19315467

1. Clara stops for 10 minutes to catch up with a friend.

Answers

Answer:

Clara has speed of 80m/min

Explanation:

Clara was jogging at 600 m in 5 minutes. She stopped suddenly which reduced her velocity and then she waited for 10 minutes so that her friends comes near her. She stopped to catch her friend. During this 10 minutes the velocity of Clara is zero. She started to walk again at a slower speed of 80m/min.

ALOT OF POINTS PLZ HURRYQQQQQQQQQQQQQQQQQQQQQQQQQQQQQQQQQQQQQQQQQQQQQQQQQQQQQQQQQQQQQQQQQQQQQQQQQWhat does Newton's third law say about why momentum is conserved in collisions?
A: Equal Forces act in equal times, so the change in momentum for both objects must be equal.
B: Unequal forces act for unequal times, so the change in momentum for both objects must be unequal.
C: Equal forces act for unequal times, so the change in momentum for both objects must be equal.
D: Unequal forces act for equal times, so the change in momentum for both objects must be equal.

Answers

Answer:

A.) Equal Forces act in equal times, so the change in momentum for both objects must be equal.

(Hope this helps! Btw, I am the first to answer.)

At a certain location, wind is blowing steadily at 10 m/s. Determine the mechanical energy of air per unit mass and the power generation potential of a wind turbine with 60-m-diameter blades at that location. Take the air density to be 1.25 kg/m3. Cengel, Yunus; Cengel, Yunus. Thermodynamics: An Engineering Approach (p. 98). McGraw-Hill Higher Education. Kindle Edition.

Answers

Answer:

1767Kw

Explanation:

Velocity of wind = 10 m/s

diameter of the blades= 60m

ρ= air density = 1.25 kg/m3

Acceleration due to gravity= 9.81 m/s^2

Mechanical energy of the wind can be calculated using the expression below

Energy= (e*m)

= ρ V A e............eqn(1)

Where A= area

ρ= air density

e= wind energy per unit mass of air

e= (v^2)/2..........eqn(2)

If we substitute the values into eqn (2) we have

e= [(10)^2]/2

=50J/Kg

But Area=A= (πd^2)/4

Area= ( π× 60^2)/4

Area=2827.8m^2

If we input substitute the values into eqn (1) we have

Energy= 1.25 ×10 × 50×2827.8

=1767145.7W

We can convert to kilo watt

=1767145.7W/ 1000

= 1767Kw

Hence, the mechanical energy of air per unit mass and the power generation potential of a wind turbine is 1767Kw

A 1 m3tank containing air at 10oC and 350 kPa is connected through a valve to another tank containing 3 kg of air at 35oC and 150 kPa. Now the valve is opened, and the entire system is allowed to reach thermal equilibrium with the surroundings, which are at 20oC. Determine the volume of the second tank and the final equilibrium pressure of air.

Answers

Answer:

- the volume of the second tank is 1.77 m³

- the final equilibrium pressure of air is 221.88 kPa ≈ 222 kPa

Explanation:

Given that;

[tex]V_{A}[/tex] = 1 m³

[tex]T_{A}[/tex] = 10°C = 283 K

[tex]P_{A}[/tex] = 350 kPa

[tex]m_{B}[/tex] = 3 kg

[tex]T_{B}[/tex] = 35°C = 308 K

[tex]P_{B}[/tex] = 150 kPa

Now, lets apply the ideal gas equation;

[tex]P_{B}[/tex] [tex]V_{B}[/tex] = [tex]m_{B}[/tex]R[tex]T_{B}[/tex]

[tex]V_{B}[/tex] = [tex]m_{B}[/tex]R[tex]T_{B}[/tex] / [tex]P_{B}[/tex]

The gas constant of air R = 0.287 kPa⋅m³/kg⋅K

we substitute

[tex]V_{B}[/tex] = ( 3 × 0.287 × 308) / 150

[tex]V_{B}[/tex] = 265.188 / 150  

[tex]V_{B}[/tex] = 1.77 m³

Therefore, the volume of the second tank is 1.77 m³

Also, [tex]m_{A}[/tex] =  [tex]P_{A}[/tex][tex]V_{A}[/tex] / R[tex]T_{A}[/tex] = (350 × 1)/(0.287 × 283) = 350 / 81.221

[tex]m_{A}[/tex]  = 4.309 kg

Total mass, [tex]m_{f}[/tex] = [tex]m_{A}[/tex] + [tex]m_{B}[/tex] = 4.309 + 3 = 7.309 kg

Total volume [tex]V_{f}[/tex] = [tex]V_{A}[/tex] + [tex]V_{B}[/tex]  = 1 + 1.77 = 2.77 m³

Now, from ideal gas equation;

[tex]P_{f}[/tex] =  [tex]m_{f}[/tex]R[tex]T_{f}[/tex] / [tex]V_{f}[/tex]

given that; final temperature [tex]T_{f}[/tex] = 20°C = 293 K

we substitute

[tex]P_{f}[/tex] =  ( 7.309 × 0.287 × 293)  / 2.77

[tex]P_{f}[/tex] =  614.6211119 / 2.77

[tex]P_{f}[/tex] =  221.88 kPa ≈ 222 kPa

Therefore, the final equilibrium pressure of air is 221.88 kPa ≈ 222 kPa

Other Questions
Two spherical waves with the same amplitude, A, and wavelength, ?, are spreading out from two point sources S1 and S2 along one side of a barrier. The two waves have the same phase at positions S1 and S2. The two waves are superimposed at a position P. If the two waves interfere constructively at P what is the relationship between the path length difference dx=d2-d1 and the wavelength. If the two waves interfere destructively at P, what is the relationship between the path length difference and the wavelength? ? QuestionDrag each tile to the correct location on the image.Match the correct description with each visual aid.should we spate our website?meINthaygodYour Money-Ndevaluation of whethera company needs toimprove itsexplanationofa complexThoreau Lewiscomparison oftwo authors'writing stylesdescription of theEmpire StateBuilding'sexplanation of howto best save andmanage moneydescription of the livesof giant pandas in theirnatural habitats Theodore Roosevelt by Shari Lyn Zuber Commonlit Jane and Peter leave their home traveling opposite directions on a straight road. Peterdrives 15 mpb faster than Jane. After 3 hours, they are 225 miles apart. What is Jane's ratein miles per hour? A mandatory health insurance plan costs $4,000. One worker earns $24,500 in employment income and $500 in investment income. Another worker earns $48,000 in employment income and $2,000 in investment income. A third worker earns $68,000 in employment income and $7,000 in investment income. A premium-based system would cost each worker $4,000. A wage tax-based system would cost each worker 8.5 percent of wages. An income tax-based system would cost each worker 8 percent of income. For each worker, calculate the cost of the insurance as a share of total income.Worker 1 Worker 2 Worker 3E=employment income $24,500 $48,000 $68,000I= investment income $500 $2,000 $7,000P=premium cost of insurance $4,000 $4,000 $4,000Premium % of income= P/(E+I)W= Wage tax cost of insurance = 0.085xEwage tax cost % income= W/(E+I)T= income tax cost = 0.080x(E+I)Income tax cost as % of income= T/(e+I)Which of the payment systems above would impose the larger burden on those with incomes under $25,000: a plan financed via premiums, via the income tax, or via a payroll tax?Which of the plans above would be fairer?Which of the preceding questions can you answer using positive economics? For which of the preceding questions must you use can you ans please ????????????????????????????????????????? why was the hebrew old testament expanded into four sections when it was translated into greek by the jews of alexandria, egypt? PHP is whitespace insensitive. It doesn't matter how many whitespace characters you have in a row. One whitespace character is the same as many such characters. True False A sample of air from a home is found to contain 4.3 ppm of carbon monoxide. This means that if the total pressure is 735 torr, then the partial pressure of CO is ________ torr.3.2 ? 10^31.7 ? 1083.25.9 ? 1033.2 ? 10^-3 if 500 additional storage space and 11 less rental limit were available, the maximum profit will be $3,890 and the optimal solution will change.T/F provide one concluding statement and two recommendations on the issue if human rights violation to communities You believe that there might be a curvilinear relation between ball inflation and accuracy. In order to test that, you estimate several models. Based on the output below, you would define the Afinal population regression equation as: t Stat p-level 0.000 Intercept Inflation 81.311 Intercept Inflation Inflation Coefficients Standard Error MODEL A 29.109 1.382 -1.181 MODEL B 50.296 0.619 -4.941 0.102 0.150 0.004 MODEL C 51.756 1.231 -5.354 0.319 0.185 0.026 -0.001 0.001 0.000 0.000 0.000 Intercept Inflation Inflation Inflation 7.118 0.000 0.000 0.178 Select one: O a. Accuracy: = Bo + BlInflation; + B2Inflation? +B3Inflation + i O O b. Accuracyi = Bo + BiInflationi + Ei C. Accuracyi = bo +b Inflationi + b2Inflation d. Accuracyi = Bo + BiInflationi + B2Inflation + i e. Accuracyi = bo + b Inflationi O O of the four types of relationships identified by psychologist constance ahrons, _____ were those who had minimal contact with each other and who became bitter and angry when they did interact. at what velocity will a 300.w motor pull a mass if it applies a force of 13.9n Derive the state-variable equations for the system that is modeled by the following ODEs where {eq}\alpha, w,{/eq} and {eq}z{/eq} are the dynamic variable and {eq}v{/eq} is the input{eq}0.4 \dot \alpha-3w+\alpha=0 \\ 0.25 \dot z+4z-0.5zw=0 \\ \ddot w+6\dot w+0.3 w^3-2\alpha=8v{/eq} which change(s) in the gastrointestinal tract can contribute to reduced nutrient absorption? How do the poems of Longfellow and Lowell reflect romantic goals and writing? Use examples from the poems what would be the current in a solenoid, in amps, that is 1.0 m long, with 11,725 turns, that generates a magnetic field of 0.6 tesla? alculate the grxn using the following information. 2 hno3(aq) no(g) 3 no2(g) h2o(l) grxn = ? gf (kj/mol) -110.9 87.6 51.3 -237.1 whole blood collected for dna-typing purposes must be placed in a vacuum containing the preservative